1
$\begingroup$

Let $\kappa$ be an uncountable cardinal.
Given a set of S of cardinality $\kappa$, I want to construct a chain {$S_\lambda : \lambda \in \kappa$ } such that:
1) Each $S_\lambda$ is a proper subset of S with cardinality $\kappa$ and
2) For any two ordinals $\alpha < \beta$, $S_\alpha - S_\beta$ has cardinality $\kappa$

Now, using axiom of choice this is easy.
Simply take $\kappa$ many copies of S and index them using some well ordering of $\kappa$.
There is a bijection between this collection and S.
Now just remove one copy at a time and use the bijection to get the corresponding subsets of S. This gives us the chain we want.

My question is, can we always obtain such a chain in ZF without choice ?

$\endgroup$
5
  • 6
    $\begingroup$ Are we using here the usual ZF definition of "cardinal" as "an ordinal not in bijection with any lesser ordinal"? If so, then $\kappa$ automatically has a well ordering. $\endgroup$ Sep 30, 2019 at 3:05
  • 2
    $\begingroup$ @Nate: That is not the usual definition. However, in order to make sense of $\lambda\in\kappa$ we pretty much have to assume that $\kappa$ is indeed a well-ordered cardinal. $\endgroup$
    – Asaf Karagila
    Sep 30, 2019 at 5:27
  • 3
    $\begingroup$ Note that if we intend for $\kappa$ to not necessarily be well-orderable (and taking into account the other comments to make the statement make sense) this is definitely false in general, as $S$ may not have any proper subsets of the same cardinality. $\endgroup$
    – Wojowu
    Sep 30, 2019 at 8:09
  • 3
    $\begingroup$ If, as previous comments indicate, $\kappa$ is supposed to be a well-ordered cardinal, then the answer is yes, because it's provable in ZF (without choice) that every infinite ordinal $\alpha$ admits a bijection to $\alpha\times\alpha$. Indeed, if you know this with choice, then it follows without choice, because you can find the required bijection in Gödel's constructible universe $L$. $\endgroup$ Sep 30, 2019 at 12:25
  • 1
    $\begingroup$ Thanks for the answers in the comments above. Why the votes to close, though ? It's not a super-advanced question, but from the comments , it looks like there are some non-trivial issues here. $\endgroup$
    – Anindya
    Oct 1, 2019 at 21:33

0

Your Answer

By clicking “Post Your Answer”, you agree to our terms of service and acknowledge you have read our privacy policy.

Browse other questions tagged or ask your own question.